Вы находитесь на странице: 1из 23

Fundamentals of Investing, 12e (Smart/Gitman/Joehnk)

Chapter 11 Bond Valuation


11.1 Learning Goal 1
1) The real rate of interest is the risk free rate minus the inflation
premium.
Answer: TRUE
2) The risk premium component of a bond's market interest rate is
related to the characteristics of the particular bond and its issuer.
Answer: TRUE
3) The risk-free rate of return considers the expected rate of inflation.
Answer: TRUE
4) Actions by the Federal Reserve can keep the risk-free rate below
the rate of inflation, at least temporarily.
Answer: TRUE
5) Municipal bonds usually have higher yields than bonds issued by
the U. S. Government.
Answer: FALSE
6) The higher a bond's Moody's or Standard & Poor's rating, the higher
its yield.
Answer: FALSE
7) Changes in the inflation rate have a direct and pronounced effect on
market interest rates.
Answer: TRUE
8) The required return on a bond is equal to
A) the real rate of return plus a risk premium plus an expected inflation
premium.
B) the real rate of return plus the coupon rate plus an inflation rate.
C) the risk-free rate plus a risk premium plus an expected inflation
premium.
D) the real rate plus a risk premium.
Answer: A
9) The risk-free rate of return is equal to the
A) real rate plus a risk premium.
B) required return minus the inflation premium.
C) real rate plus the inflation premium.
D) required return minus the real rate.
Answer: C

A) I and III only


B) II and IV only
C) I, II and IV only
D) I, III and IV only
Answer: D
13) Which of the following risks are included in the risk premium?
I.
interest rate risk
II. liquidity risk
III. financial risk
IV. purchasing power risk
A) I and II only
B) II and III only
C) III and IV only
D) I and IV only
Answer: B
14) Which one of the following will tend to cause domestic interest
rates to rise?
A) an increase in the money supply
B) a decrease in the rate of inflation
C) a decrease in the federal budget deficit
D) an increase in interest rates overseas
Answer: D
15) The single most important factor that influences the behavior of
market interest rates is
A) inflation.
B) business profits.
C) the supply of new bonds.
D) the stock market.
Answer: A
16) Which one of the following statements concerning interest rates is
correct?
A) A decrease in the money supply will cause interest rates to decline.
B) A federal budget surplus will cause interest rates to decline.
C) Economic expansions will cause interest rates to decline.
D) Rising interest rates in foreign countries will cause U.S. interest
rates to decline.
Answer: B
11.2 Learning Goal 2
1) The relationship between the rate of return and the time to maturity
of similar-risk securities is known as the term structure of interest rates.
Answer: TRUE

10) Rank the following taxable bonds from lowest yielding to highest
yielding.
I.
U.S. Treasury bonds
II. corporate bonds
III. agency bonds
A) I, II, III
B) II, I, III
C) III, II, I
D) I, III, II
Answer: D

2) A yield curve depicts the term structure of interest rates for similarrisk securities.
Answer: TRUE

11) Which of the following factors will tend to cause the risk free rate to
rise?
I.
an increase in the money supply
II. an increase in the federal budget deficit
III. an increase in the level of economic activity
IV. falling rates in foreign markets
A) I, II, III only
B) II, III only
C) I and IV
D) I, II, III and IV
Answer: B

5) A normal yield curve is flat or downward sloping.


Answer: FALSE

12) Which of the following factors influence short-term interest rates on


government securities?
I.
Federal Reserve actions
II. interest rate risk
III. expected future inflation
IV. the real rate of return

3) Predicting the direction of interest rate movements is relatively easy.


Answer: FALSE
4) Treasury bond yields are commonly used as the basis for yield
curves because they are low risk and homogeneous in nature.
Answer: TRUE

6) A downward sloping yield curve (short-term rates are higher than


long-term rates) often precedes a recession.
Answer: TRUE
7) The real rate of return is the same for all maturities.
Answer: TRUE
8) According to the expectations hypothesis, if investors anticipate
higher rates of inflation in the future, the yield curve will be
downsloping.
Answer: FALSE
9) According to the liquidity preference theory, borrowers should pay a
higher interest rate for long-term borrowing than for short-term
borrowing.

Answer: TRUE
10) A down-sloping yield curve indicates that interest rates are about to
rise.
Answer: FALSE
11) A steep yield curve is generally considered a bullish sign for bonds.
Answer: TRUE

20) The liquidity preference theory supports ________ yield curves.


A) upward sloping
B) flat
C) humped
D) downward sloping
Answer: A

12) The yield curve depicts the relationship between a bond's yield to
maturity and its
A) duration.
B) term to call.
C) term to maturity.
D) volatility.
Answer: C

21) The market segmentation theory holds that


A) an increase in demand for long-term borrowings leads to an
inverted yield curve.
B) expectations about the future level of interest rates is the major
determinant of the shape of the yield curve.
C) the yield curve reflects the maturity preferences of financial
institutions and investors.
D) the shape of the yield curve is always downsloping.
Answer: C

13) An inverted yield curve


A) means that long-term bonds are yielding more than short-term
bonds.
B) exists when intermediate-term bonds have higher yields than either
short-term or long-term bonds.
C) rewards long-term investors for the additional risk they are
assuming.
D) generally results from actions by the Federal Reserve to control
inflation.
Answer: D

22) Market segmentation theory explains the typical upward sloping


shape of yield curves as a function of
A) normally greater demand for long-term bonds than for short-term
notes.
B) normally greater demand for short term notes than for long-term
bonds.
C) expectations that inflation will be higher in the future than it is now.
D) the greater liquidity of short-term notes as compared to long-term
bonds.
Answer: B

14) The expectations hypothesis states that investors


A) require higher long-term interest rates today if they expect higher
inflation rates in the future.
B) expect higher long-term interest rates because of the lack of liquidity
for long-term bonds.
C) require the real rate of return to rise in direct proportion to the length
of time to maturity.
D) normally expect the yield curve to be downsloping.
Answer: A

23) At any given time, the yield curve is affected by


I.
lender preferences.
II. inflationary expectations.
III. liquidity preferences.
IV. short- and long-term supply and demand conditions.
A) I and IV only
B) II, III and IV only
C) I, II and III only
D) I, II, III and IV
Answer: D

15) According to expectations theory if the 1 year interest is 3% this


year and expected to be 5% next year, the 2 year interest rate should
be approximately
A) 8%.
B) 5%.
C) 4%.
D) 3%.
Answer: C
16) According to the expectations hypothesis, investors' expectations
of decreasing inflation will result in
A) a downward-sloping yield curve.
B) an upward-sloping yield curve.
C) a flat yield curve.
D) a humped yield curve.
Answer: A
17) Downward sloping or flat yield curves often indicate
A) a recession in the near future.
B) an economic expansion in the near future.
C) higher inflation in the near future.
D) a weaker dollar in the foreign exchange markets.
Answer: A

24) Which of the following theories is consistent with yield curves


sloping upward most of the time?
I.
market segmentation theory.
II. expectations theory.
III. liquidity preference theory.
IV. theory of evolution.
A) I and III only
B) II, III and IV only
C) I, II and III only
D) I, II, III and IV
Answer: A
25) If the yield curve begins to rise sharply, it is usually an indication
that
A) stocks are offering low returns as the economy enters a recession.
B) inflation rates have peaked and are about to decline.
C) bond prices are expected to increase.
D) inflation is starting to increase, or is expected to do so in the near
future.
Answer: D

18) Long-term bonds are ________ than short-term bonds.


A) less risky
B) more liquid
C) subject to more uncertainty
D) less sensitive to interest rate changes
Answer: C

26) Evidence indicates that the theory of interest rates with the most
predictive power is
A) market segmentation theory.
B) expectations theory.
C) liquidity preference theory.
D) a combination of expectations, market expectations and liquidity
preference.
Answer: D

19) When compared to the yield curve for Treasury securities, the yield
curve for corporate securities should
A) slope in the opposite direction.
B) be similar in shape but higher.
C) be similar in shape but lower.
D) be nearly identical.
Answer: B

27) If inflation is expected to increase significantly, cautious


bondholders should
A) expect interest rates to rise.
B) expect a flat yield curve for the intermediate-term.
C) buy long-term bonds today.
D) move to the short-end of the yield curve.
Answer: D

11.3 Learning Goal 3


1) If a bond's yield to maturity is lower than its coupon rate, the bond
will sell at a discount.
Answer: FALSE
2) A bond's discount or premium will tend to increase as the bond
approaches its maturity date.
Answer: FALSE
3) The price of a bond is equal to the present value of the bond's future
cash flows.
Answer: TRUE
4) As a bond approaches its maturity date, its price necessarily
approaches par value.
Answer: TRUE
5) The longer the time to maturity, the less sensitive a bond's price will
be to changes in interest rates.
Answer: FALSE

13) Which of the following are needed to determine the appropriate


value of a bond?
I.
required rate of return
II. time to maturity
III. frequency of interest payments
IV. coupon rate
A) II and III only
B) III and IV only
C) II, III and IV only
D) I, II, III and IV
Answer: D
14) A $1,000 par value, 12-year annual bond carries a coupon rate of
7%. If the current yield of this bond is 7.995%, its market price to the
nearest dollar is
A) $876.
B) $925.
C) $1,075.
D) $1,125.
Answer: A
11.4 Learning Goal 4

6) A significant portion of a coupon bond's total return is derived from


the reinvestment of the interest payments.
Answer: TRUE
7) A bond has a coupon rate of 6%, matures in 6 years, and currently
sells for $1,000 (par value). Therefore the yield to maturity is also 6%.
Answer: TRUE
8) The price of a bond with an 8% coupon rate paid semi-annually, a
par value of $1,000, and fifteen years to maturity is the present value
of
A) 15 payments of $40 at 6 month intervals plus $1,000 received at the
end of the fifteenth year.
B) 15 payments of $80 at 6 month intervals plus $1,000 received at the
end of the fifteenth year.
C) 30 payments of $40 at 6 month intervals plus $1,000 received at the
end of the fifteenth year.
D) 30 payments of $80 at 1 year intervals plus $1,000 received at the
end of the 30th year.
Answer: C
9) What is the current price of a 9%, $1,000 annual coupon bond that
has eighteen years to maturity and a yield to maturity of 9.631%?
A) $898
B) $935
C) $942
D) $947
Answer: D
10) What is the coupon rate of an annual bond that has a yield to
maturity of 8.5%, a current price of $942.32, a par value of $1,000 and
matures in thirteen years?
A) 7.67%
B) 7.75%
C) 8.33%
D) 8.50%
Answer: B
11) What is the current price of a $1,000, 6% coupon bond that pays
interest semi-annually if the bond matures in ten years and has a yieldto-maturity of 7.1325%?
A) $567
B) $920
C) $1,030
D) $1,080
Answer: B
12) What is the yield-to-maturity of a $1,000, 7% semi-annual coupon
bond that matures in 2 years and currently sells for $997.07?
A) 6.87%
B) 7.04%
C) 7.16%
D) 7.31%
Answer: C

1) There is normally an indirect relationship between the coupon rate


of a bond and the bond's yield.
Answer: FALSE
2) Generally speaking, short-term bonds have lower yields than longterm bonds.
Answer: TRUE
3) A basis point is 1/10 of 1%.
Answer: FALSE
4) Bond yields are set by the bond issuer.
Answer: FALSE
5) The required return defines the yield at which a bond should be
trading and serves as the discount rate in the bond valuation process.
Answer: TRUE
6) A bond's yield to maturity is equal to the internal rate of return of its
cash flows.
Answer: TRUE
7) A bond's current yield is equal to the interest payment divided by par
value.
Answer: FALSE
8) The actual return earned on a bond is highly dependent upon the
reinvestment rate of the coupons.
Answer: TRUE
9) Yield-to-call assumes a bond is called on the last possible date.
Answer: FALSE
10) The greater of the yield-to-call or the yield-to-maturity is used as
the appropriate indicator of value.
Answer: FALSE
11) Yield to call is a useful measure for bonds selling at a premium, but
not for bonds selling at a discount.
Answer: TRUE
12) Which one of the following statements is correct concerning bond
investors?
A) Aggressive investors want to lock in high interest rates.
B) Aggressive investors purchase bonds when they believe interest
rates will rise.
C) Conservative investors seek capital gains.
D) Conservative investors buy bonds when interest rates are high.
Answer: D
13) If you are an income-oriented investor and you feel that interest
rates are relatively high and will decline in the future, you should
purchase
A) zero-coupon, long-term bonds.

B) long-term, non-callable bonds.


C) short-term, zero-coupon bonds.
D) long-term, freely callable bonds.
Answer: B
14) Which of the following statements concerning the current yield is
correct?
A) It is of great interest to aggressive bond investors seeking capital
gains.
B) It is of great interest to conservative bond investors seeking current
income.
C) It shows the rate of return an investor will receive by holding a bond
to maturity.
D) It can be determined by dividing interest income by the par value of
a bond.
Answer: B
15) A bond is most likely to be called
A) when investors must reinvest at lower rates.
B) when the bond sells at a large discount.
C) when market yields are close to coupon rates.
D) when investors can reinvest at higher rates.
Answer: A
16) Which of the following statements are correct concerning yield-tomaturity (YTM)?
I.
YTM considers both interest income and price appreciation.
II. YTM assumes the bond is called at the earliest possible date.
III. YTM is a compounded rate of return.
IV. YTM assumes all interest payments are reinvested at the YTM
rate.
A) I and IV only
B) I, III and IV only
C) II, III and IV only
D) I, II and III only
Answer: B
17) Which one of the following statements is true about a $1,000, 6%
annual coupon bond that is selling for $1,012?
A) The current yield is less than 6%.
B) The current yield is 6%.
C) The yield-to-maturity is greater than 6%.
D) The yield-to-maturity is 6%.
Answer: A
18) Dylan purchased 20 GIA Inc. bonds in 2008 when the yield to
maturity was 12.5%. Because of falling interest rates she had to
reinvest the coupon payments at 8%, 6%, 4% and finally, 3%. The
internal rate of return on her investment will be
A) greater than the coupon rate but less than the original yield to
maturity.
B) less than the original yield to maturity.
C) greater than the original yield to maturity.
D) the reinvestment rate has no effect on the internal rate of return.
Answer: B
19) Yield to call on a bond with a coupon rate of 8% paid semiannually, 10 years to maturity, a par value of $1,000 and a selling price
of $1,071, callable after 5 years at $1,010 is
A) 3.5%.
B) 6.49%.
C) 7.0%.
D) 8.16%.
Answer: B
20) The current yield on a bond is most similar to
A) the discount rate on a Treasury Bill.
B) the effective annual rate on a certificate of deposit.
C) the dividend yield on a stock.
D) the internal rate of return if the bond is held to maturity.
Answer: C
21) The actual return on a bond is dependent upon which of the
following?
I.
the coupon rate
II.
the reinvested interest rate
III. any changes in par value

IV. any changes in market price


A) I, II and III only
B) II, III and IV only
C) I, III and IV only
D) I, II and IV only
Answer: D
22) The reinvestment rate assumption is more important
I.
the longer the time to maturity.
II. the shorter the time to maturity.
III. the higher the coupon rate.
IV. the lower the coupon rate.
A) I and III
B) I and IV
C) II and III
D) II and IV
Answer: A
23) Yield-to-call is
A) commonly used for bonds with deferred-call provisions.
B) calculated using the time to call and the par value of the bond.
C) based solely on the call premium and ignores interest payments.
D) always less than the yield-to-maturity.
Answer: A
24) Hunter bought a bond with an 8% coupon rate for $1,100 and sold
it one year later for $1,150. His holding period return was
A) 11.8%.
B) 11.3%.
C) 13.0%.
D) 7.27%.
Answer: A
25) Nathan bought a zero coupon bond in 2003 for $485.19. In 2013
he redeemed it for $1,000. His internal rate of return on this
investment was
A) 206.1%.
B) 20.6%.
C) 7.5%.
D) 0.00%.
Answer: C
26) The yield-to-maturity (YTM) approach fails to consider which of the
following risks?
I.
reinvestment risk
II. price or market risk
A) I only
B) II only
C) Both I and II
D) Neither I nor II
Answer: C
11.5 Learning Goal 5
1) Bond duration refers to the remaining life of a bond.
Answer: FALSE
2) An increase in interest rates has a negative effect on bond prices
and a positive effect on the reinvestment of coupons.
Answer: TRUE
3) The duration of a bond portfolio is the weighted average of the
durations of the individual bonds included in the portfolio.
Answer: TRUE
4) With exception of zero coupon bonds, a bond's duration is always
shorter than its time to maturity.
Answer: TRUE
5) When yield swings are relatively small, a bond's duration is a viable
predictor of its price volatility.
Answer: TRUE
6) A bond portfolio manager believes that interest rates are about to
increase. Given this belief, the manager should buy long duration
bonds and sell short duration bonds.
Answer: FALSE

7) When the weighted-average duration of an investor's bond portfolio


is exactly equal to the investor's desired investment horizon, then the
bond portfolio is said to be immunized.
Answer: TRUE
8) As applied to bonds, duration refers to
A) the average maturity of a diversified portfolio of corporate bonds.
B) the point in the life of a bond when its price exactly offsets its
reinvestment risk.
C) the average price and annual reinvestment rate of return for a bond.
D) the point in the life of a bond when its yield-to-maturity equals its
expected yield.
Answer: B
9) Based on the concept of bond duration, which one of the following
statements is correct?
A) Lower coupons result in shorter durations.
B) Longer maturities mean shorter durations.
C) Higher yields (YTMs) lead to longer durations.
D) Longer durations mean greater volatility.
Answer: D
10) Which one of the following bonds would have a duration that
exactly matches its time to maturity?
A) discount bond
B) premium bond
C) zero-coupon bond
D) U.S. Treasury bond
Answer: C
11) Which of the following statements concerning duration are correct?
I.
Duration is a weighted-average life of a bond.
II. The Macaulay duration considers the timing of a bond's cash
flows.
III. The Macaulay duration uses the YTM of a bond to discount the
cash flows.
IV. For coupon bonds, duration will be less than the actual time to
maturity.
A) I, II and III only
B) II, III and IV only
C) I, III and IV only
D) I, II, III and IV
Answer: D
12) Which of the following risks can be essentially eliminated by
immunizing a bond portfolio?
I.
default risk
II. price risk
III. reinvestment risk
IV. liquidity risk
A) I, II and III only
B) II and III only
C) II, III and IV only
D) I, II, III and IV
Answer: B
13) The duration of a bond will increase as the time to maturity
________ and/or as the YTM on the bond ________.
A) increases; increases
B) increases; decreases
C) decreases; increases
D) decreases; decreases
Answer: B
14) A $1,000, 7% annual coupon bond matures in three years. The
bond is currently priced at $974.23 and has a YTM of 8.0%. What is
the Macaulay duration?
A) 1.95 years
B) 2.60 years
C) 2.81 years
D) 3.00 years
Answer: C
15) A portfolio consists of three bonds as follows.

What is the duration of the bond portfolio?


A) 7.12 years
B) 8.07 years
C) 8.69 years
D) 11.4 years
Answer: A
16) A bond matures in 30 years, has a 20 year duration and a yield to
maturity of 9.32%. The market interest rate has increased by 0.47%.
The modified duration is
A) 9.4 years.
B) 14.1 years.
C) 18.29 years.
D) 27.44 years.
Answer: C
17) A bond matures in 30 years, has a 20 year duration and a yield to
maturity of 9.32%. The change in the level of the market interest rate is
0.47%. The modified duration is ________ and the percentage change
in price is ________.
A) 9.4 years, -.47%
B) 14. years, 4.7%
C) 18.29 years., -8.6%
D) 18.29 years., 8.6%
Answer: C
18) The mathematical link between a bond's price and interest rate
changes is the
A) Macaulay duration.
B) modified duration.
C) yield to market.
D) yield-to-call.
Answer: B
19) A bond has a YTM of 6.5%, a modified duration of 16.9 years, a
duration of 18 years and a 30 year maturity. By what percentage will
the bond's price change if market interest rates increase by 0.75%?
A) -0.750 percent
B) +0.750 percent
C) +12.675 percent
D) -12.675 percent
Answer: D
20) If the bond market undergoes a large change in yield (for example,
more than 100 basis points), then a bond's duration will
A) understate both the price appreciation when rates fall and the price
decline when rates increase.
B) overstate both the price appreciation when rates fall and the price
decline when rates increase.
C) overstate the price appreciation when rates fall and understate the
price decline when rates increase.
D) understate the price appreciation when rates fall and overstate the
price decline when rates increase.
Answer: D
21) The practical application of bond portfolio immunization to
investors is that immunization
A) allows aggressive traders to eliminate the price effects caused by
interest rate changes.
B) allows investors to derive a specified rate of return from bond
investments for a given investment horizon.
C) eliminates the possibility of losing money due to a company
defaulting on its bond payments.
D) allows investors to passively manage their bond portfolio once it is
initially immunized.
Answer: B
11.6 Learning Goal 6
1) In building a bond ladder, an investor invests an equal amount in a

series of bonds with staggered maturities.


Answer: TRUE

Fundamentals of Investing, 12e (Smart/Gitman/Joehnk)


Chapter 12 Mutual Funds and Exchange-Traded Funds

2) A major advantage of passive bond strategies is low transaction


costs.
Answer: TRUE

12.1 Learning Goal 1

3) When conducting a tax swap, an investor must use identical issues


in order for the swap to be allowed by the IRS.
Answer: FALSE
4) Buying bonds in anticipation of an expected decline in interest rates
is a risky strategy.
Answer: TRUE
5) Once a bond portfolio is initially immunized, an investor should
maintain the portfolio as it is until the end of the investment horizon.
Answer: FALSE
6) The main purpose of a bond ladder is to
A) lessen the effects of changes in interest rates.
B) achieve the highest level of capital gains possible.
C) maintain a highly liquid portfolio.
D) offset the effects of bond duration.
Answer: A
7) Active bond trading strategies include
I.
buy and hold.
II. trading on forecasted interest rate behavior.
III. bond ladders.
IV. bond swaps.
A) I and III
B) II and IV
C) I, II and III
D) II, III and IV
Answer: B
8) Reasons for using a bond ladder strategy include
I.
typically higher rates on long-term bonds.
II. uncertainty concerning future interest rates.
III. lower tax rates on bonds held to maturity.
IV. reducing the amount of time spent managing the bond portfolio.
A) I and III
B) II and IV
C) I, II and III
D) I, II and IV
Answer: D
9) Some common types of bond swaps are
I.
tax swaps.
II. yield pickup swaps.
III. substitution swaps.
IV. credit default swaps.
A) I and III
B) II and IV
C) I, II and III
D) I, II and IV
Answer: C
10) Suppose you sell the 10-year, A-rated 7 percent bonds you own,
which are yielding 8 percent, and replace them with an equal amount
of 10-year, A-rated 8 percent bonds that are priced to yield 9 percent.
In this situation, you are executing
A) an immunization deal.
B) a yield pickup swap.
C) a laddered bid.
D) a spread bid.
Answer: B
11) In a tax swap, a bond investor typically
A) sells an issue which has a capital loss and replaces it with a
comparable security.
B) sells an issue that has a capital gain and replaces it with a
comparable security.
C) swaps a lower-yielding security for a higher-yielding security.
D) swaps a higher-yielding security for a lower-yielding security.
Answer: A

1) Mutual funds provide a simplified means of diversifying a portfolio.


Answer: TRUE
2) When an investor buys shares in a mutual fund, he or she becomes
a part owner of a portfolio of securities.
Answer: TRUE
3) Mutual funds rank second only to banks as being the largest
financial intermediary in the United States.
Answer: FALSE
4) The mutual fund market is dominated by funds that invest in
diversified bond portfolios.
Answer: FALSE
5) The transfer agent for a mutual fund physically safeguards the
securities being bought and sold by that firm.
Answer: FALSE
6) Mutual funds tend to outperform the market.
Answer: FALSE
7) The mutual fund, and not the investor, is responsible for all income
taxes on capital gains and dividends earned by the fund.
Answer: FALSE
8) Which one of the following statements concerning mutual funds is
correct?
A) The selection of individual securities remains with the mutual fund
investor.
B) Mutual funds were first created in the 1980s.
C) The mutual fund industry is the largest financial intermediary in the
United States.
D) Mutual funds are generally highly concentrated portfolios.
Answer: C
9) Which of the following are advantages offered by mutual funds?
I.
professional portfolio management
II. dividend reinvestment
III. consistent returns in excess of the overall market rate of return
IV. modest capital outlay for investors
A) I and II only
B) I and IV only
C) II, III and IV only
D) I, II and IV only
Answer: D
10) One drawback of investing in mutual funds is the
A) annual management fee.
B) lack of liquidity of fund shares.
C) amount required for the initial investment.
D) lack of information on the performance of the fund.
Answer: A
11) Which one of the following statements is correct concerning
players in the mutual fund industry?
A) Security analysts and traders work for the management company.
B) Normally a bank serves as the custodian.
C) The management company maintains the shareholder records.
D) The mutual fund shareholders are the owners of the management
company.
Answer: B
12) During the 7 year market cycle of 2006-2012, in which category of
funds did a majority of funds outperform the market average?
A) large cap funds
B) small cap funds
C) asset allocation funds
D) In no category did a majority of funds outperform the market
average.
Answer: D

13) Which of the following statements best describes the legal


organization of mutual funds?
A) Funds are organized as a single entity that handles all functions
such as custody and investment decisions.
B) Funds split their basic functions such as record keeping and
investment decisions among two or more companies.
C) Funds are owned by the company that manages them.
D) A distributor keeps track of investment and redemption requests
from shareholders and maintains other shareholder records.
Answer: B
14) To operate as a regulated investment company and enjoy the
related tax benefits, a mutual fund must annually distribute to its
shareholders
A) half of its realized capital gains, and interest and dividend income.
B) none of its realized capital gains, but all of its interest and dividend
income.
C) all of its realized capital gains, and at least 90 percent of its interest
and dividend income.
D) all of its realized capital gains and interest and dividend income.
Answer: C
15) Mutual fund investors delegate all of the following decisions to the
fund's managers EXCEPT
A) which companies and industries to invest in.
B) when to buy and sell individual stocks.
C) how many securities to hold in the portfolio.
D) how to allocate investments among different classes of assets such
as stocks, bonds, cash and real estate.
Answer: D
16) Nearly all mutual funds operate as regulated investment
companies. This means that
A) they are no-load funds.
B) portfolio decisions are mandated by government authorities.
C) they do not pay taxes on their income.
D) their investments are guaranteed by the FDIC.
Answer: C

10) Hedge funds are subject to the same regulations and disclosure
requirements as mutual funds.
Answer: FALSE
11) Most exchange-traded funds are index funds.
Answer: TRUE
12) Investors who buy or sell exchange-traded funds will do so at a
price based on the closing price for the day.
Answer: FALSE
13) Mutual fund fees are disclosed in the fund prospectus.
Answer: TRUE
14) The net asset value is the price per share an investor will pay to
acquire shares in a no-load, open-end fund.
Answer: TRUE
15) The longer you intend to hold a fund, the more willing you should
be to accept a front-end load charge in exchange for lower annual
management and 12(b)-1 fees.
Answer: TRUE
16) An open-end investment company
A) is involved in all trades of its shares.
B) sells shares at a discounted NAV price.
C) trades like a stock on the exchanges.
D) has a set number of shares.
Answer: A
17) The net asset value of a mutual fund increased from $12.03 to
$13.53, but its price per share increased by only $1.26. This
information indicates that the fund
A) paid out $1 in capital gains.
B) paid out $1 in dividends.
C) is a closed-end fund.
D) is an open-end fund.
Answer: C

12.2 Learning Goal 2


1) Investors purchase shares in an open end mutual fund directly from
the fund.
Answer: TRUE
2) The purchase price of a closed-end mutual fund is equivalent to the
net asset value of the fund.
Answer: FALSE
3) The discount or premium on a closed-end mutual fund can be as
much as 25 percent.
Answer: TRUE
4) Trading in closed-end investment companies takes place between
investors in the open market.
Answer: TRUE
5) As a general rule, the time to buy a closed-end mutual fund is when
the fund's premium is approximately 5 percent higher than its past
average.
Answer: FALSE
6) A closed end fund is selling at a premium when the NAV exceeds
the market price.
Answer: FALSE
7) A closed-end fund with an NAV of $9.60 and a market price of
$10.25 is selling at a premium of 6.8%.
Answer: TRUE
8) Closed-end funds are required to provide a prospectus to all new
shareholders as long as the fund is trading at a premium.
Answer: FALSE
9) Like ordinary stocks, exchange traded funds (ETFs) can be sold
short.
Answer: TRUE

18) Which of the following characteristics apply to closed-end mutual


funds?
I.
unlimited number of outstanding shares
II. transactions between shareholders
III. Market prices may be higher or lower than NAV.
IV. Fund will repurchase shares at any time.
A) I and IV only
B) II and III only
C) I, II and III only
D) II, III and IV only
Answer: C
19) Which of the following characteristics apply to exchange traded
funds (ETFs)?
I.
unlimited number of outstanding shares
II. typically track the performance of some index
III. Market prices may be higher or lower than NAV.
IV. May invest in the whole index or use a sample of securities to
track the index as closely as possible.
A) I and II only
B) II and III only
C) I, II and III only
D) I, II and IV only
Answer: D
20) Closed-end funds are
A) less liquid than open-end funds.
B) best purchased when they are selling at a premium.
C) purchased directly from the funds' manager.
D) traded at NAV.
Answer: A
21) Which one of the following statements concerning ETFs is correct?
A) ETFs are based solely on U.S. indexes.
B) The ETF based on the Standard & Poor's 500 Index is priced at
1/10 the value of that index.
C) Spiders are based on the DJIA.

D) The ETF based on the Dow is priced at 1/10 of the value of the
DJIA.
Answer: B
22) Which of the following statements is(are) correct concerning
exchange-traded funds (ETFs)?
I.
You can buy and sell ETFs any time during trading hours.
II. ETFs are actively managed.
III. ETFs have high portfolio turnover rates.
IV. ETFs rarely distribute any capital gains.
A) I, II and IV only
B) I and IV only
C) II and III only
D) I, III and IV only
Answer: B
23) The commission charged when shares of an open-end mutual fund
are purchased is called a
A) management fee.
B) back-end load.
C) front-end load.
D) 12(b)-1 fee.
Answer: C
24) Back-end loads
A) are charged when an investor buys their mutual fund shares.
B) are charged if an investor sells his or her shares within the first few
years.
C) were designed to help no-load funds cover their marketing
expenses.
D) encourage short-term trading.
Answer: B
25) One characteristic of 12(b)-1 charges is that they are payable
A) only in years that the mutual fund shows an increase in net asset
value.
B) each year regardless of the performance of the mutual fund.
C) only during the first year the fund is owned.
D) only when shares in the fund are sold.
Answer: B
26) Two mutual funds are quoted as follows.

Given these quotes, which one of the following is true?


A) Both funds are load funds.
B) Fund A is a no-load fund.
C) Fund B is a no-load fund.
D) Both funds are no-load funds.
Answer: C
27) A type of fund that invests in real estate and/or mortgages is known
as a
A) REIT.
B) ETF.
C) sector fund.
D) hedge fund.
Answer: A
28) Which type of fund is always passively managed?
A) a closed-end fund
B) a growth fund
C) a value fund
D) an index fund
Answer: D

funds?
I.
They are highly regulated.
II. They hedge all positions to limit risks.
III. Management and other fees are extremely low compared to other
types of funds.
IV. Access is limited to institutions and high net worth or high income
individuals.
A) I, II and III only
B) II and IV only
C) IV only
D) I, II, III and, IV
Answer: C
31) Investors in hedge funds have the legal status of
A) shareholders.
B) limited partners.
C) general partners.
D) trustees.
Answer: B
12.3 Learning Goal 3
1) The primary objective of growth mutual funds is capital appreciation
with a high level of current income.
Answer: FALSE
2) Morningstar, a leading. mutual fund industry publication, classifies
funds by the number of shareholders and the total amount of money
invested in the fund.
Answer: FALSE
3) Index funds merely attempt to match the performance of some
benchmark, not to outperform it.
Answer: TRUE
4) The maximum average maturity of the holdings within a money
market account must be 6 months or less.
Answer: FALSE
5) Socially responsible funds only hold stocks of companies that meet
the fund's ethical guidelines.
Answer: TRUE
6) A unit investment trust
A) engages in short-term trading within a particular sector.
B) offers a low-cost, diversified portfolio.
C) is an unmanaged portfolio of securities.
D) is used only for fixed-income securities.
Answer: C
7) Risk-seeking investors seeking maximum capital appreciation with
little, if any current income, should invest in
A) value funds.
B) growth funds.
C) aggressive growth funds.
D) equity-income funds.
Answer: C
8) An aggressive growth mutual fund is least likely to purchase a stock
A) with a high P/E ratio.
B) with a high anticipated rate of growth.
C) of an unseasoned firm.
D) with a high dividend yield.
Answer: D

29) Performance fees based on profits earned by the fund are typical
of
A) hedge funds.
B) exchange traded funds.
C) closed-end investment companies.
D) open end mutual funds.
Answer: A

9) Aggressive mutual funds often employ investing strategies such as


I.
short selling.
II. margin trading.
III. option trading.
IV. hedging.
A) I and III only
B) II and III only
C) I, II and IV only
D) I, II, III and IV
Answer: D

30) Which of the following statements is(are) correct concerning hedge

10) Value funds seek stocks

I.
with low dividend yields.
II.
with potential for growth.
III. with low P/E ratios.
IV. of newly discovered firms.
A) I and III only
B) II and III only
C) II, III and IV only
D) I, II, III and IV
Answer: B
11) The primary objective of an equity-income fund is
A) capital gains.
B) current income with capital preservation.
C) potentially high capital gains with limited income.
D) high risk-return trade-offs.
Answer: B
12) Which type of mutual fund consists of both stocks and bonds with a
combined objective of current income and long-term capital gains?
A) equity-income
B) balanced
C) value
D) bond
Answer: B
13) One characteristic of bond funds is the
A) requirement of a minimum initial investment of $5,000 or more.
B) high anticipated short-term growth potential.
C) fluctuation in value in response to changing interest rates.
D) extremely aggressive trading approach.
Answer: C
14) One advantage gained by investing in a bond fund rather than in
individual bonds is the
A) diversification among issuers.
B) most bond funds outperform their benchmarks.
C) immunity from interest rate changes.
D) guarantee that the bonds will be held to maturity to avoid market
fluctuations.
Answer: A
15) Which of the following are advantages of bond funds over
individual bonds?
I.
greater liquidity
II. maturities matched to the investor's time horizon
III. automatic reinvestment of interest payments
IV. diversification
A) I and III only
B) I, III and IV only
C) I, II and IV only
D) II, III and IV only
Answer: B
16) Investors who seek triple (federal, state, and local) tax-free income
should invest in ________ bond funds.
A) convertible
B) indexed
C) mortgage-backed
D) single-state municipal
Answer: D
17) Compared to yields on general purpose money funds, the yields on
tax-exempt money funds are
A) about the same.
B) 5 to 10 percent higher.
C) 5 to 10 percent lower.
D) 20 to 30 percent lower.
Answer: D
18) Government securities money funds are structured to eliminate
A) interest rate risk.
B) inflation risk.
C) default risk.
D) market risk.
Answer: C
19) A fund that is designed to match the performance of a measure

such as the S & P 500 or the Russell 2000 is called a(n)


A) index fund.
B) targeted fund.
C) sector fund.
D) block fund.
Answer: A
20) One characteristic of most index funds is that such funds typically
A) produce a large dollar amount of realized capital gains every year.
B) have a very low-cost structure with respect to management fees
and transaction fees.
C) charge high front-end loads.
D) are designed to "beat the market."
Answer: B
21) Ashley believes that the price of gas and oil is about to rise and
energy company profits will follow. She should invest in
A) an asset allocation fund.
B) an emerging market fund.
C) a sector fund.
D) an aggressive growth fund.
Answer: C
22) Socially responsible funds are distinguished from other mutual
funds because they
A) invest only in over-the-counter stocks.
B) do not charge any sales commission or management fees.
C) invest only in companies that meet specified moral, ethical, or
environmental standards.
D) will sell their shares only to investors who sign a statement saying
they do not smoke tobacco or use alcohol.
Answer: C
23) Funds that invest in a portfolio of companies from the same or
closely related industries are known as
A) aggressive growth funds.
B) sector funds.
C) emerging market funds.
D) asset allocation funds.
Answer: B
24) One type of mutual fund spreads investors' money across equity
markets, bond markets, and money markets. Moreover, as market
conditions change, the amount of money invested in each market
sector will change. This type of mutual fund is known as a(n)
A) socially responsible fund.
B) fiscally responsible fund.
C) growth-and-income fund.
D) asset allocation fund.
Answer: D
25) Which one of the following statements is correct concerning
international funds?
A) A devaluation of the dollar causes returns on foreign investments to
improve from a U.S. perspective.
B) International funds are considered low-risk investments.
C) Balance-of-trade positions do NOT affect the rate of return from a
U.S. perspective.
D) Technically, global funds can only invest in foreign securities.
Answer: A
12.4 Learning Goal 4
1) Automatic investment plans makes it easier for investors to save
money.
Answer: TRUE
2) To participate in an automatic investment plan, investors must allow
the investment company to have access to a bank account or their
paycheck.
Answer: TRUE
3) Dividends and capital gains earned by mutual fund investors are
taxed when the shares are redeemed.
Answer: FALSE
4) The conversion privilege offered by many mutual funds enables

investors, within three days, to convert their shares back to cash in an


amount equal to the original cost of the investment.
Answer: FALSE

12.5 Learning Goal 5


1) Mutual funds are used extensively as retirement investments.
Answer: TRUE

5) The conversion privilege allows investors to switch from one fund to


another without fees if an only if the funds are managed by the same
company such Fidelity or Vanguard.
Answer: TRUE

2) Investors should select mutual funds that match their personal


investment goals and provide the services they desire.
Answer: TRUE

6) Systematic withdrawal plans from mutual funds usually require a


minimum investment of at least $100,000.
Answer: FALSE

3) Investors should never use mutual funds to invest in areas where


they would not buy individual stocks.
Answer: FALSE

7) The ability to automatically buy additional fund shares using the


dividend income generated by the fund is called a(n)
A) automatic investment plan.
B) automatic reinvestment plan.
C) systematic withdrawal plan.
D) conversion plan.
Answer: B

4) Funds that consistently earn above average rates of return also tend
to charge higher sales commissions.
Answer: FALSE

8) Typical services offered by mutual funds include:


I.
Automatic reinvestment plans.
II. Automatic investment plans.
III. Automatic fund conversion as investment goals change.
IV. Automatic withdrawal plans
A) I and III only
B) II and IV only
C) I, II and IV only
D) I, II, III and IV
Answer: C

6) An investor who wants to use mutual funds as a storehouse of value


should invest in
A) high-yield corporate bond funds and growth funds.
B) growth funds and equity-income funds.
C) equity-income funds and long-term bond funds.
D) money funds and short-term bond funds.
Answer: D

9) Which of the following is a default option for most mutual funds?


A) Automatic reinvestment of dividends and interest income.
B) Automatic investment of a fixed sum each month.
C) Automatic conversion from aggressive to conservative funds as
clients approach retirement.
D) Automatic withdrawal of a fixed amount each month.
Answer: A
10) Automatic reinvestment plans
A) are a good way to avoid taxes on dividends and capital gains.
B) may involve exceptionally high transaction fees.
C) do not allow for the purchase of fractional shares.
D) are an excellent way to accumulate wealth through disciplined
investing.
Answer: D
11) Investors interested in predictable cash flow from their investments
should consider funds that offer
A) conversion privileges.
B) systematic withdrawal plans.
C) automatic reinvestment plans.
D) automatic investment plans.
Answer: B
12) Mutual funds often report returns as the growth of $10,000 over a
period of time. These returns assume that
A) all dividends and capital gains are reinvested.
B) all dividends and capital gains are withdrawn.
C) all dividends and capital gains are reinvested after deductions for
income taxes.
D) the investor contributes money to the fund on a regular basis
through an automatic investment plan.
Answer: A
13) The conversion privilege provided by mutual fund families allows
investors to
A) move from one fund to another without incurring any capital gains
tax liability.
B) be more aggressive since they can re-allocate their funds when
market conditions change.
C) re-allocate their funds at any time as long as they pay an additional
sales load on the transferred funds.
D) move from one fund family to another once every six months.
Answer: B

5) Mutual funds and exchanged traded funds are available to meet the
goals of both aggressive and conservative investors.
Answer: TRUE

7) Investors use mutual funds for which of the following reasons?


I.
to accumulate wealth
II. to minimize risk
III. as a speculative vehicle
IV. as a storehouse of value
A) I and II only
B) II, III and IV only
C) I, II and IV only
D) I, II, III and IV
Answer: D
8) Tax laws allow investors to hold mutual funds in which of the
following types of retirement plans?
I.
ordinary IRAs
II. Roth IRAs
III. 401-K plans
IV. Keogh plans
A) I and II only
B) II, III and IV only
C) I, II and IV only
D) I, II, III and IV
Answer: D
9) Which of the following accurately describe reasons for investing in
mutual funds?
I.
to effectively control the timing of capital gains for tax purposes
II. to achieve portfolio diversification at a reasonable cost
III. to invest in unfamiliar sectors or geographic regions
IV. to outperform the market
A) I and IV only
B) II and III only
C) I, II and III only
D) I, II, III and IV
Answer: B
10) Reasons to invest in mutual fund include
I.
a wide range of services such automatic reinvestment and
systematic withdrawal plans.
II. minimizing the time and effort spent choosing securities.
III. rates of return that consistently beat the market averages.
IV. participation in a variety of tax sheltered and tax deferred
retirement programs.
A) I and II only
B) II, III and IV only
C) I, II and IV only
D) I, II, III and IV
Answer: C

11) A mutual fund is generally more tax efficient when it has a


________ turnover rate and a ________ dividend yield.
A) low; low
B) low; high
C) high; low
D) high; high
Answer: A
12) Investors seeking tax-exempt income should invest in
A) index funds.
B) government bond funds.
C) municipal bond funds.
D) money market funds.
Answer: C
12.6 Learning Goal 6
1) Unless the investor specifically requests otherwise capital gains are
normally distributed as cash payments.
Answer: FALSE
2) Both realized and unrealized capital gains from mutual fund
investments are currently taxable for individual income tax purposes.
Answer: FALSE
3) Only realized capital gains from mutual fund investments are
currently taxable for individual income tax purposes.
Answer: TRUE
4) Both the performance of the overall stock market as well as the
skills of the mutual fund manager affect the performance of a mutual
fund.
Answer: TRUE
5) Holding period returns are normally used only for investment periods
of one year or less.
Answer: TRUE
6) Last year, Sue purchased a closed-end mutual fund that was trading
at $42 and had an NAV of $38. Sue sold the fund today when the NAV
is $44 and the market price is $43. The fund paid $1 in dividends over
the past year. What is the Sue's holding period return?
A) 4.8%
B) 7.1%
C) 11.6%
D) 18.4%
Answer: A
7) Investors are generally well advised to avoid mutual funds with
A) highly rated fund managers.
B) low fees and high tax efficiency.
C) consistently poor historical performance.
D) good performance in both up and down markets.
Answer: C
8) Returns on exchange traded funds may come from
I.
capital gains.
II. dividends.
III. increases in the fund's premium.
IV. decreases in the fund's discount.
A) I and II only
B) III and IV only
C) I, II and III only
D) I, II, III and IV
Answer: A
9) Which of the following are sources of income for an open-end
mutual fund?
I.
dividend and interest income
II. change in the discount or premium
III. capital gains
IV. change in NAV
A) I and III only
B) I, II and III only
C) I, III and IV only
D) II, III and IV only
Answer: C

10) Income distributed by a mutual fund from which one of the


following sources receives a preferential tax rate of 15%?
A) dividends on common stock
B) interest on bonds
C) dividends from most preferred stocks
D) dividends from REITs
Answer: A
11) Investment companies can inflate the percentage of funds that
outperformed their benchmark by
A) including unrealized capital gains.
B) closing funds that underperformed their benchmark.
C) using a different fiscal year than the benchmark.
D) assuming that investors automatically add to their holdings.
Answer: B
12) A mutual fund's net asset value is determined by
A) demand for the fund based on the fund's performance.
B) the fund manager's reputation.
C) the value of the securities held by the fund.
D) all of the above.
Answer: C
13) Income from ________ is exempt from federal income taxes?
A) money market funds
B) municipal bond funds
C) government bond funds
D) REITs
Answer: B
14) Last year at this time, a mutual fund had an NAV of $13.20 per
share. Over the past year the fund paid dividends of $0.70 per share
and had a capital gains distribution of $1.20 per share. What is the
holding period return assuming that the current NAV is $14.42?
A) 13.2%
B) 14.4%
C) 21.6%
D) 23.6%
Answer: D
15) Many funds advertise their returns as the amount to which $10,000
invested in the fund would have grown to over 5 or 10 years. These
results assume that
A) all dividends, interest and capital gains on a pre-tax basis are
reinvested in the fund.
B) all dividends and capital gains less a 15% deduction for taxes are
reinvested in the fund.
C) all dividends and interest are reinvested in the fund.
D) no dividends or capital gains are reinvested in the fund.
Answer: A
16) Information for ABC Fund is

The market-based holding period return for ABC Mutual Fund is


A) -19.1%.
B) -14.6%.
C) 0.3%.
D) 9.4%.
Answer: B
17) What is the average annual compound rate of return for a fund with
the following values?

Answer: A

A) 10.5%
B) 10.8%
C) 25.6%
D) 32.3%
Answer: A
18) Mutual fund investors are primarily exposed to ________ and
________ risks.
A) market; financial
B) market; inflation
C) business; financial
D) business; inflation
Answer: B

Fundamentals of Investing, 12e (Smart/Gitman/Joehnk)


Chapter 13 Managing Your Own Portfolio
13.1 Learning Goal 1
1) Before analyzing needs and objectives, investors should first
construct a portfolio.
Answer: FALSE
2) A conservative asset allocation would rely heavily on bonds and
short-term securities.
Answer: TRUE
3) Utility stocks are often suitable for low-risk, current-income-oriented
portfolios.
Answer: TRUE
4) When developing an asset allocation scheme, it is best to weight
each type of asset equally.
Answer: FALSE
5) An investment portfolio should be built around the needs of the
individual investor.
Answer: TRUE
6) Asset allocation focuses on selecting specific securities within an
asset class.
Answer: FALSE
7) The most important factor in allocation schemes is the investor's
age.
Answer: FALSE
8) An investor's portfolio should only contain securities that are suitable
to the investor's goals and needs.
Answer: TRUE
9) Marti is 31 years old and is saving for retirement. Which one of the
following portfolio allocations might best suit her situation if she is
willing to accept a fair amount of risk in exchange for long-term capital
appreciation?
A) 60% bonds, 15% money funds and 25% real estate
B) 5% money funds, 10% bonds and 85% growth stocks
C) 25% bank CDs, 40% corporate bonds, 15% money market, 20%
value stocks
D) 50% mortgage bonds, 5% money market, 45% municipal bonds
Answer: B
10) Asset allocation should focus on
A) the investor's financial and family situation.
B) selection of individual securities within an asset class.
C) maximization of current income.
D) maximization of short-term profits.

11) A moderate asset allocation alternative might include


I.
bonds.
II. common stocks.
III. foreign securities.
IV. options and commodities futures.
A) I and II only
B) I, III and IV only
C) I, II and III only
D) I, II, III and IV
Answer: C
12) Which one of the following provides the greatest reduction in total
risk?
A) diversification
B) asset allocation
C) security selection
D) beta reduction
Answer: B
13) The fixed-weightings approach to asset allocation
A) is based on an allocation of an equal percentage of the portfolio to
each separate asset category.
B) requires periodic rebalancing of the portfolio to maintain the desired
weights.
C) is based on periodic adjustments to category weights in response to
market changes.
D) uses stock-index futures and bond futures in a market timing
strategy.
Answer: B
14) Tactical asset allocation is most suitable for
A) young, single individuals with good incomes.
B) retirees.
C) large institutional investors.
D) investors who have already accumulated a fair amount of wealth.
Answer: C
15) An asset allocation plan should consider which of the following
factors?
I.
economic outlook
II. capital preservation
III. changing investment goals
IV. investor risk tolerance
A) II only
B) II, III and IV only
C) I, III and IV only
D) I, II, III and IV
Answer: D
16) An asset allocation plan should consider which of the following
investor characteristics?
I.
income and employment security
II. marital status
III. age and proximity to retirement
IV. social relationships and peer groups
A) II only
B) I, II and III only
C) I, III and IV only
D) I, II, III and IV
Answer: B
17) Fred and Martha are in their seventies and retired. Which one of
the following sets of portfolio statistics might best suit their situation if
their primary investment goal is current income with limited risk?
A) beta of 0.83 and a dividend yield of 6.3%
B) beta of 0.86, and a dividend yield of 4.6%
C) beta of 1.6 and a dividend yield of 6.4%
D) beta of 1.1 and a dividend yield of 5.4%
Answer: A
13.2 Learning Goal 2
1) The Dow Jones Industrial Average (DJIA) includes 500 of the largest
companies traded on U. S. exchanges.
Answer: FALSE

2) The key areas to monitor when evaluating your portfolio holdings


are the overall performance of both the economy and the financial
markets, and the returns on your investments.
Answer: TRUE
3) Once you establish a portfolio designed to achieve your investment
goals, you can relax and forget about your investments until such time
as you need the funds.
Answer: FALSE
4) Investors need to monitor economic and market activity to assess
the potential impact these factors can have on their investment
portfolios.
Answer: TRUE
5) Several indexes are available to monitor the performance of stocks,
but nothing similar is available for bonds or mutual funds.
Answer: FALSE
6) Most investment professionals consider the Dow Jones Industrial
Average (DJIA) to be the most appropriate comparative gauge for
evaluating the investment performance of a broadly based common
stock portfolio.
Answer: FALSE
7) The S&P 500 Stock Composite Index and the NASDAQ Composite
Index can be used to represent the stock market as a whole.
Answer: TRUE
8) If an investor's portfolio is comprised of a broad range of common
stocks, the best measure to use as a basis of comparison of
performance is the
A) Dow Jones Industrial Average (DJIA).
B) S&P 500 index.
C) Dow Jones Corporate Bond Index.
D) American Stock Exchange utilities index.
Answer: B

D) II, III and IV only


Answer: A
13.3 Learning Goal 3
1) The holding period return measures only the capital appreciation of
an investment.
Answer: FALSE
2) Returns for periods greater than one year should be measured
using the internal rate of return.
Answer: TRUE
3) Holding period return (HPR) captures total return performance by
considering current income and capital gains and is most appropriate
for holding periods of one year or less.
Answer: TRUE
4) A rational investor will require the same return from a corporate
security as from a government security.
Answer: FALSE
5) investors need to consider the effects of taxes and transaction fees
when measuring portfolio returns.
Answer: TRUE
6) If the holding period return (HPR) of an investment is 20 percent
before taxes for a nine month period, an investor in the 30 percent tax
bracket would have an after-tax HPR of 14 percent.
Answer: TRUE
7) The calculation of returns on options and futures must consider
income as well as capital gains.
Answer: FALSE
8) Only capital gains that have been realized should be included in the
measurement of a portfolio's return over a given period of time.
Answer: FALSE

9) The S & P 500 Index is an appropriate benchmark for


A) diversified portfolios of large company stocks.
B) portfolios diversified among several asset classes such as stocks,
bonds, and real estate.
C) diversified portfolios with a mix of large, small, and mid-cap stocks.
D) diversified portfolios of mid-cap and small company stocks.
Answer: A

9) For a stock investment, the dividend yield is calculated by


A) dividing a stock's annual cash dividend by its price.
B) dividing a stock's price by its annual cash dividend.
C) multiplying a stock's semi-annual dividend by two.
D) dividing the annual change in the stock's price plus its annual
dividend amount by the beginning of the year price.
Answer: A

10) The best index to assess the performance of a portfolio diversified


among several asset classes such as stocks, bonds and real estate is
A) the Lipper Index.
B) the NYSE Composite Index.
C) the Value Line Index.
D) No suitable index exists.
Answer: D

10) The holding period return for mutual funds should be based on
A) net asset value exclusively.
B) dividend income exclusively
C) capital gains distributions exclusively
D) capital gains distributions and dividends.
Answer: D

11) Lipper indexes are to assess the performance of


I.
equity funds.
II. bond funds.
III. money market funds.
IV. Real Estate Investment Trusts (REITs).
A) I and II only
B) I and III only
C) I, II and III only
D) I, II, III and IV
Answer: A
12) The holding period return (HPR) of one's portfolio should be
compared to investment goals
I.
to assess whether the proper rate of return is being earned for the
risk involved.
II. to be sure one's portfolio is outperforming the S&P 500 Index.
III. to isolate any problem investments.
IV. to determine when to change benchmarks from the S&P 500 to
the NASDAQ Composite Index.
A) I and III only
B) II and IV only
C) I, II and III only

11) The holding period return (HPR)


A) reflects only capital gains and losses for investment periods of one
year or less.
B) calculates the annual dividend yield on stocks or current interest
yield on bonds.
C) is the most appropriate measure of returns for an investment period
exceeding one year.
D) can be used to determine the actual total return on stocks, bonds,
and other investments for periods of one year or less.
Answer: D
12) To compute the holding period return on a bond investment, the
investor should divide the purchase price of the bond into
A) any increase or decrease in the bond's price.
B) the annual coupon payment.
C) the bond's yield to maturity.
D) coupon payments received plus or minus any change in the bond's
price.
Answer: D
13) Juan's investment portfolio was valued at $125,640 at the
beginning of the year. During the year, Juan received $603 in interest
income and $298 in dividend income. Juan also sold shares of stock

and realized $1,459 in capital gains. Juan's portfolio is valued at


$142,608 at the end of the year. All income and realized gains were
reinvested. No funds were contributed or withdrawn during the year.
What is the amount of income Juan must declare this year for income
tax purposes?
A) $0
B) $901
C) $2,360
D) $19,328
Answer: C

20) An investor in the 25% marginal tax bracket purchased a bond for
$983, received $85 in interest, and then sold the bond for $955 after
holding it for six months. The tax rate for capital gains with holding
periods in excess of one year is 15%. What are the pre-tax and posttax holding period returns?
A) 5.8%; 4.3%
B) 6.0%; 4.5%
C) 5.8%; 4.5%
D) 6.0%; 4.3%
Answer: A

14) On January 1, Stacy's portfolio was valued at $96,534. During the


year Stacy received $3,285 in interest and $4,100 in dividends. She
also sold one stock at a gain of $850. The value of the portfolio on
December 31 of the same year was $113,201. At the end of June,
Stacy withdrew $5,000 from the portfolio. What is the holding period
return for the year?
A) 25.1%
B) 25.8%
C) 26.5%
D) 27.2%
Answer: C

21) Investors who wish to minimize the effect of taxes on their


investment returns should try to avoid
A) dividend paying stocks.
B) short-term capital gains.
C) long-term capital gains.
D) municipal bonds.
Answer: B

15) Six months ago, Suzanne purchased a stock for $28 a share.
Today she sold the stock at a price of $32 a share. During the time she
owned the stock, she received a total of $1.30 in dividends per share.
What is her holding period return?
A) 16.6%
B) 18.9%
C) 33.2%
D) 37.8%
Answer: B
16) Ten months ago, Junior purchased a stock for $14 a share. The
stock pays a quarterly dividend of $0.50 per share. Today, Junior sold
the stock for $15 a share. What is his holding period return?
A) 10.0%
B) 10.7%
C) 16.7%
D) 17.9%
Answer: D
17) Ten years ago, Taylor purchased 444.44 shares in a mutual fund
for $22.50 per share. He has never made an additional investment in
this fund, but because of reinvested dividends and capital gains, he
now owns 1,200 shares with a net asset value of $25.88 per share.
Ignoring taxes, his compound average annual rate of return (IRR) is
A) 10.0%.
B) 12%.
C) 21%.
D) 31%.
Answer: B
18) Tim purchased a stock ten months ago for $14 a share, received a
$1 dividend per share last month, and sold the stock today for $16 per
share. Tim has a marginal tax rate of 30%. Both capital gains for
securities held more than one year and dividend income is taxed at
15%. What is Tim's after-tax holding period return?
A) 14.1%
B) 15.9%
C) 16.1%
D) 18.2%
Answer: C
19) On February 19, 2004, Angela purchased 100 shares of ABC stock
at a total cost of $1,712.50. She received a total of $125.00 in
dividends and sold the stock today, February 22, 2005. Her net
proceeds from the sale are $1,892.40. Angela has a combined state
and federal marginal tax rate of 32%. Her combined state and federal
tax rate on both her capital gains in excess of one year and her
dividend income is 18%. What is Angela's after-tax holding period
return on her investment in ABC stock?
A) 11.0%
B) 12.1%
C) 13.2%
D) 14.6%
Answer: D

22) Maria purchased $5,000 of no-load mutual fund shares just over a
year ago. She received $136 in dividend income and $201 in long-term
capital gains distributions. Today she sold her shares for $5,062. Maria
is in the 25% marginal tax bracket. Capital gains with holding periods
in excess of one year and dividend income are taxed at 15%. What is
Maria's after-tax holding period return?
A) 6.0%
B) 6.6%
C) 6.8%
D) 8.0%
Answer: C
23) On January 1, Tim's portfolio was valued at $432,098. During the
year Tim received $10,563 in interest and $15,060 in dividends. He
also sold stock at a net loss of $12,870 and used the proceeds to
purchase another stock. Tim did not contribute any more funds nor
withdraw any funds during the year. On December 31 of the same
year, Tim's portfolio was valued at $398,189. What is the holding
period return for the year?
A) -5.3%
B) -4.9%
C) -2.1%
D) -1.9%
Answer: D
13.4 Learning Goal 4
1) Sharpe's measure of portfolio performance adjusts for risk by
dividing total portfolio return by the portfolio beta.
Answer: FALSE
2) Sharpe's measure of portfolio performance compares the risk
premium on a portfolio to the portfolio's standard deviation of return.
Answer: TRUE
3) Sharpe's measure is a measure of the risk premium per unit of total
risk.
Answer: TRUE
4) Jensen's measure of portfolio performance compares the risk
premium on a portfolio to the portfolio's beta.
Answer: FALSE
5) Sharpe's measure, Treynor's measure, and Jensen's measure all
focus on non-diversifiable risk.
Answer: FALSE
6) Sharpe measures total risk while Treynor and Jensen measure only
systematic risk.
Answer: TRUE
7) A Jensen measure of 2.5% means that a security earned 2.5% more
than the overall market.
Answer: FALSE
8) Portfolio revision is the ongoing process of systematically studying
the issues in the portfolio and selling certain issues and purchasing
others as the means of maintaining a portfolio that best meets the

investor's objectives.
Answer: TRUE
9) Which of the following are reasons to consider selling an investment
that is currently in a portfolio?
I.
The investment has met the original objective.
II. Better investment opportunities currently exist.
III. The outlook for the investment has improved.
IV. The investment has not met expectations and no change is
expected.
A) I, II and IV only
B) I, III and IV only
C) I, II and III only
D) I, II, III and IV
Answer: A
10) A problem investment
A) requires immediate attention.
B) is one you would buy if you did not already own it.
C) is defined as any investment with unrealized losses.
D) should be left alone and given time to correct itself.
Answer: A
11) Allison's portfolio has an expected return of 14% and a standard
deviation of of 20%. Brianna's portfolio has an expected rate of return
of 11% and a standard deviation of 12%. The risk-free rate is 3%.
According to the Sharpe measure,
A) Allison has the better portfolio.
B) Brianna has the better portfolio.
C) The portfolio's are equally desirable.
D) The answer depends on Allison and Brianna's risk tolerance.
Answer: B
12) Ella owns a stock with a beta of 1.34 and a standard deviation of
16.4%. The stock has a total return of 14.8%. The market risk premium
is 8.5%, while the return on the market portfolio was 12.0%. What is
the value of Sharpe's measure for Ella's portfolio?
A) 0.21
B) 0.38
C) 0.69
D) 0.90
Answer: C
13) Sharpe's measure of portfolio performance compares the risk
premium on a portfolio to
A) a broad-based market index such as the S&P 500 index.
B) the portfolio's standard deviation of return.
C) the portfolio's beta.
D) the prevailing risk-free rate of return.
Answer: B
14) The Sharpe's measure for Jane Smith's investment portfolio is
0.40, while the Sharpe's measure for the market is 0.30. This
information suggests that Smith's portfolio
A) exhibits superior performance because its risk premium per unit of
risk is above that of the market.
B) exhibits poor performance because its risk premium per unit of risk
is below that of the market.
C) is inadequately diversified, and more securities should be added to
the portfolio in order to bring it in line with the market.
D) is overly diversified, and some securities should be sold to bring the
portfolio in line with the market.
Answer: A
15) Phil has a portfolio with a 13.2% total return. The beta of the
portfolio is 1.48 and the standard deviation is 13%. Currently, the riskfree rate of return is 4% and the overall market has a total return of
11%. What is the value of Treynor's measure for Phil's portfolio?
A) 2.1%
B) 6.2%
C) 7.1%
D) 8.9%
Answer: B
16) Allison's portfolio has an expected return of 14% and a beta of
1.37. Brianna's portfolio has an expected rate of return of 11% and a
beta of 1. The risk-free rate is 3%. According to the Treynor measure,

A) Allison has the better portfolio.


B) Brianna has the better portfolio.
C) The portfolio's are equally desirable.
D) The answer depends on Allison and Brianna's risk tolerance.
Answer: C
17) A portfolio has a total return of 14.5%, a beta of 1.54, and a
standard deviation of 17.6%. If the risk free rate is 4.5% and the
market return is 10.2%, then Treynor's measure of this portfolio's
performance is
A) 2.8%.
B) 3.7%.
C) 6.5%.
D) 9.4%.
Answer: C
18) A stock has a total return of 16.4%, a standard deviation of 14.5%
and a beta of 1.63. The market rate of return is 12.4%, while the
market's Treynor measure is 6.3. What is the value of the Treynor
measure of this portfolio?
A) 2.5%
B) 6.3%
C) 18.4%
D) 27.6%
Answer: B
19) Treynor's measure of portfolio performance focuses on
A) nondiversifiable risk.
B) diversifiable risk.
C) total risk.
D) the standard deviation of the portfolio.
Answer: A
20) Which of the following measures is based on the capital asset
pricing model?
A) Only Sharpe's measure
B) Only Treynor's measure
C) Only Jensen's measure
D) Both Treynor's and Jensen's measures
Answer: D
21) A portfolio has a total return of 10.5%, a beta of 0.72 and a
standard deviation of 6.3%. The risk free rate is 3.8%, the market
return is 12.4%. Jensen's measure of this portfolio's performance is
A) 0.5%.
B) 4.3%.
C) 7.9%.
D) 9.3%.
Answer: A
22) Which one of the following statements is correct if a portfolio has a
Jensen measure of return of zero?
A) The portfolio has a total return of zero percent.
B) The portfolio earned exactly its expected return on a risk-adjusted
basis.
C) The portfolio outperformed the market on a risk-adjusted basis.
D) The market provides a better return on a risk-adjusted basis.
Answer: B
23) Allison's portfolio has an expected return of 14% and a beta of
1.37. Brianna's portfolio has an expected rate of return of 11% and a
beta of 1. The risk-free rate is 3% and the expected rate of return on
the market is 12%. According to the Jensen's measure,
A) Allison has the better portfolio.
B) Brianna has the better portfolio.
C) the portfolios are equally desirable.
D) the answer depends on Allison and Brianna's risk tolerance.
Answer: B
24) Which of the following statements about Jensen's measure are
correct?
I.
Through its use of the capital asset pricing model, Jensen's
measure automatically adjusts for market return.
II. In general, the higher the Jensen's measure, the better a portfolio
has performed.
III. Jensen's measure is referred to as alpha.
IV. A positive Jensen's measure indicates an investment has

underperformed the market on a risk-adjusted basis.


A) I and IV only
B) I, II and III only
C) II and III only
D) I, III and IV only
Answer: B
25) The process of selling certain issues in a portfolio and purchasing
new ones to replace them is known as
A) portfolio revision.
B) market timing.
C) red herring baiting.
D) dollar cost averaging.
Answer: A

high.
C) to equal the performance of market averages at the lowest dollar
cost.
D) to sell as markets decline and buy as they begin to rise.
Answer: B
12) When using a constant dollar plan,
A) gains from the speculative portion of the portfolio are transferred to
the safe portion of the portfolio.
B) the amount of money in the speculative portion of the portfolio
should decline over time.
C) the percentage of funds in the speculative portion of the portfolio
should increase over time.
D) the ratio of safe funds to speculative funds remains constant over
time.
Answer: A

13.5 Learning Goal 5


1) Formula plans are high-risk investment strategies that attempt to
benefit from cyclical price movements.
Answer: FALSE
2) Dollar cost averaging is a formula plan to purchase the same
number of shares of stock at regular intervals of time.
Answer: FALSE
3) Dollar cost averaging is a formula plan which automatically causes
investors to purchase more shares when the price is low and purchase
fewer shares when the price is high.
Answer: TRUE
4) Successful dollar cost averaging depends on the investor's skill in
timing the market.
Answer: FALSE
5) Investors who use formula plans believe that they have above
average ability to time the market and pick successful investments.
Answer: FALSE
6) A constant-ratio plan requires an investor to periodically rebalance
the portfolio.
Answer: TRUE
7) A constant ratio plan allows for speculative gains while limiting
potential losses.
Answer: TRUE
8) Which one of the following statements concerning formula plans is
correct?
A) The use of subjective judgment is important to the routine
administration of most formula plans.
B) Formula plans are based on the adherence to a mechanical set of
rules with regard to when to buy and/or sell.
C) The objective of most formula plans is to maximize profits.
D) Securities with very stable prices are best suited to nearly all
formula plans.
Answer: B
9) Dollar cost averaging is a procedure by which an investor
A) buys more stock as its price increases.
B) times investments in order to buy low and sell high.
C) invests a fixed dollar amount in a security at fixed intervals.
D) maintains a constant ratio of conservative and aggressive
investments.
Answer: C
10) Which one of the following statements is correct concerning dollar
cost averaging plans?
A) Dollar cost averaging is an active trading strategy.
B) Dollar cost averaging is a short-term trading strategy.
C) The goal of dollar cost averaging is current dividend income.
D) The goal of dollar cost averaging is long-term capital appreciation.
Answer: D
11) The general theory of dollar cost averaging is
A) to time the market to take advantage of low stock prices.
B) to buy more stock when prices are low and less when prices are

13) The formula plan that requires maintaining a target dollar


investment in the speculative portion of an investor's portfolio is the
A) most passive of all the formula plans.
B) target return plan.
C) constant ratio plan.
D) constant dollar plan.
Answer: D
14) If a constant-dollar plan portfolio is profitable over the long run, the
________ in value over time.
A) conservative portion will increase
B) conservative portion will remain constant
C) aggressive portion will decrease
D) entire portfolio will remain constant
Answer: A
15) The constant-ratio plan
A) requires the establishment of trigger points for portfolio rebalancing.
B) utilizes a predetermined ratio between desired current yield and
expected capital gains.
C) strictly adheres to a buy-and-hold strategy.
D) is an attempt to time the cyclical movements of the market.
Answer: A
16) The formula plan which requires the greatest management
attention and is also the most aggressive is called the ________ plan.
A) dollar cost averaging
B) constant dollar
C) constant ratio
D) variable ratio
Answer: D
17) The theory behind the variable ratio plan is to
A) passively buy and hold a wide variety of securities.
B) time the cyclical movements of the stock market and thereby "buy
low and sell high."
C) avoid selling any security for a capital gain, and thus indefinitely
avoiding the capital gains tax.
D) keep the unit cost of the portfolio at a constant level.
Answer: B
18) Investors who who accept the random walk theory should use
A) a dollar cost averaging plan.
B) a constant dollar plan.
C) a constant ratio plan.
D) a variable ratio plan.
Answer: A
19) Which of the following is ideally suited to automatic investing
through a payroll deduction plan?
A) a dollar cost averaging plan
B) a constant dollar plan
C) a constant ratio plan
D) a variable ratio plan
Answer: A
20) Under the variable-ratio plan, additional speculative investments
are made when the ratio
A) of conservative investments to speculative investments increases
by 10%.

B) of the rate of return on the speculative investments exceeds the


overall market return by 1% or more.
C) of the realized rate of return falls below the desired rate of return by
1% or more.
D) of the value of the speculative investments to the total portfolio
value drops below a predetermined level.
Answer: D

III. the guaranteed execution within the order period


IV. the conversion to a market order
A) I and II only
B) III and IV only
C) I, II and IV only
D) II, III and IV only
Answer: C

21) An investor adopts a policy of investing in both an aggressive


mutual fund and a short-term bond fund. When the value of the
aggressive fund exceeds 65% of the portfolio value, shares of that fund
are sold such that the aggressive fund represents only 45% of the
portfolio. This is an example of a ________ plan.
A) constant-dollar
B) dollar-cost averaging
C) constant-ratio
D) variable-ratio
Answer: D

11) Suppose the shares of the Chickadee Corporation traded seven


times in the following sequence one day last week: 46, 45.88, 45.75,
45.50, 45.63, 46, 46.13. In this case, a limit order to sell at 46 would
have been executed
A) between 46 and 46.13, whereas a market order to sell could have
been executed anywhere between 45.50 and 46.13.
B) anywhere between 45.50 and 46.13, whereas a market order to sell
would have been executed only at 46.
C) only at 46, whereas a market order to sell would have been
executed at 46.13.
D) only at 46.13, and a market order to sell would have been executed
between 46 and 46.13.
Answer: A

13.6 Learning Goal 6


1) A stop-loss order guarantees that an investor's unrealized profit will
be protected.
Answer: FALSE
2) If an investor has a loss position in an investment and wants to sell
it, the best time to sell for tax purposes is when a capital gain is
available against which the loss can be applied.
Answer: TRUE
3) The maximum capital loss that can be applied to ordinary income for
income tax purposes in any one year is $3000.
Answer: TRUE
4) An investor who wants to take advantage of a temporary decline in
the price of a stock should use a limit order.
Answer: TRUE
5) Over a period of time if an investment has not met its return
objective, it should be sold.
Answer: TRUE
6) For tax purposes, it is advisable to sell stocks that have lost value
and keep stocks that have increased in value.
Answer: TRUE
7) The primary risk in using a GTC limit sell order rather than a market
order is that
A) the market price may exceed the limit price when the order is
placed.
B) the limit order may not be executed.
C) the limit order may be executed at a price above the market price.
D) the limit order expires at the end of the day and may not be
executed.
Answer: B
8) A stop loss order may not protect an investor's profits if
A) the price drops even slightly below the stop price before the order
can be executed.
B) the price enters a prolonged period of gradual decline.
C) an unexpected event cause the price to drop steeply when the
markets are closed.
D) the stop loss price is never reached.
Answer: C
9) A stop loss order may not be advisable
A) when an investor wants to protect a gain.
B) an investor wants is expecting an earnings announcement that may
either exceed or fall short of expectations.
C) a stock exhibits wide and rapid price fluctuations.
D) when an investor wants to sell in the near future but still profit from
any short-term price increases.
Answer: C
10) Which of the following are characteristics of stop-loss orders?
I.
the risk of whipsawing
II. the ability to limit downside losses

12) The two primary media for warehousing liquidity are


A) money market mutual funds and money market deposit accounts.
B) certificates of deposit and short-term bond funds.
C) certificates of deposit and long-term bond funds.
D) short-term bond funds and asset allocation funds.
Answer: A
13) Which of the following are reasons why a person may want to
warehouse liquidity?
I.
protect against total loss
II. ability to exploit future opportunities
III. capitalize on the high rates of return available on cash
IV. protect against the need to disturb the existing portfolio
A) I and II only
B) I, II and IV only
C) II, III and IV only
D) I, II, III and IV
Answer: B
14) One important tax rule concerning capital losses is that
A) capital losses are always fully deductible.
B) a maximum of $3,000 of losses in excess of capital gains can be
written off against ordinary income in any one year.
C) a maximum of $10,000 of losses in excess of capital gains can be
written off against other income in any one year.
D) capital losses are never deductible.
Answer: B
15) Which of the following are generally considered to be good
investment guidelines?
I.
Sell any security that has become riskier than anticipated.
II. Hold all securities until they produce the highest profit attainable.
III. Sell securities only if the profit can be offset with a tax loss.
IV. Sell any security that no longer meets the needs of the investor.
A) I and IV only
B) I and III only
C) I, II and IV only
D) I, II, III and IV
Answer: A
Fundamentals of Investing, 12e (Smart/Gitman/Joehnk)
Chapter 14 Options: Puts and Calls
14.1 Learning Goal 1
1) Because puts and calls derive their value from the behavior of some
other real or financial asset, they are known as derivative securities.
Answer: TRUE
2) Investors who purchase options acquire nothing more than the right
to buy or sell the shares of the underlying security.
Answer: TRUE
3) Options are created by investors.
Answer: TRUE

Answer: C
4) Puts and calls are issued by the same corporation that issued the
underlying stock.
Answer: FALSE
5) The owner a put is obliged to sell the underlying security at the
strike price on the date of expiration.
Answer: FALSE
6) Rights are call options issued to current owners of the stock and
normally expire within a short period of time.
Answer: TRUE
7) Options allow investors to speculate on price movements without a
large initial investment.
Answer: TRUE
8) Warrants are short-term options usually expiring within a year or
less.
Answer: FALSE
9) Warrants are options that are attached to bond issues to make the
bonds more attractive to investors.
Answer: TRUE
10) Rights and warrants are the riskiest types of options.
Answer: FALSE
11) Purchasers of stock options
A) own a financial asset with benefits of firm ownership.
B) have a claim on the profits of the firm issuing the underlying
securities.
C) have the obligation to buy or sell a predetermined amount of shares
at the strike price.
D) have the right to buy or sell a certain number of underlying shares.
Answer: D
12) Which one of the following statements concerning options is
correct?
A) One option covers 1,000 shares of stock.
B) A put gives the option holder the right to buy a stated amount of
securities.
C) The owner of a call is entitled to the dividends paid on the
underlying shares of stock.
D) Option holders can profit on movements of the price of the
underlying security.
Answer: D
13) An American call option gives the owner
A) the right to buy or sell the stock at the strike price on or before the
expiration date.
B) the right but not the obligation to buy the stock at the strike price on
or before the expiration date.
C) the right and the obligation to buy the stock at the strike price on or
before the expiration date.
D) the right but not the obligation to sell the stock at the strike price on
or before the expiration date.
Answer: B
14) Which of the following statements concerning options are correct?
I.
Options are derivative securities.
II. The value of an option is dependent upon the value of the
underlying security.
III. The seller of the option retains the option premium whether or not
the option is exercised.
IV. Options can provide leverage benefits.
A) II and III only
B) I, II and III only
C) I, II and IV only
D) I, II, III and IV
Answer: D
15) Which of the following is true about rights?
A) They are usually attached to bonds as a "sweetener."
B) The owner has several years in which to exercise the option.
C) They are a type of short-lived call option.
D) They are a type of short-lived put option.

16) The maker of a put or call is the


A) company which issued the underlying security.
B) person who facilitates the trade on the floor of the exchange.
C) party who writes the option.
D) party who decides whether or not the option is exercised.
Answer: C
17) Writers of option contracts
A) have a limited liability specified in the contract.
B) hope to exercise the option on favorable terms.
C) earn a commission no matter what subsequently happens to the
contract.
D) earn a profit when the option expires without being exercised.
Answer: D
18) One reason that writing options can be a viable and profitable
investment strategy is that
A) the option writer collects the quarterly dividends.
B) most options expire unexercised.
C) an option writer determines when the option is exercised.
D) an option writer can exercise the option to avoid a potential loss.
Answer: B
19) The ability to obtain a given equity position at a reduced capital
investment, and therefore magnify returns, is known as
A) leverage.
B) straddling.
C) hedging.
D) triple witching.
Answer: A
20) LEAPS are a special type of option
A) that must be exercised within six months.
B) that can only be exercised on the expiration date.
C) that cannot be exercised for at least a year after it is is purchased.
D) that may have an expiration date as long as three years.
Answer: D
21) Warrants are generally created when
A) a firm decides to execute a stock split.
B) the issuing corporation decides to sweeten a bond issue.
C) a LEAP expires and automatically converts.
D) a financial institution decides to create them based on market
conditions.
Answer: B
22) LEAPS is an acronym for
A) Lehman and Ellsworth Authority Strips.
B) Liability & Equity Asset Securities.
C) LYONS Earnings Anticipation Stocks.
D) Long-Term Equity Anticipation Securities.
Answer: D
23) Warrants
A) provide substantially less capital appreciation potential than the
underlying stock.
B) tend to be quite costly.
C) have a stipulated price and an expiration date.
D) are not traded in the secondary markets because of their low unit
costs.
Answer: C
14.2 Learning Goal 2
1) An options strike price is the stock price at which the option holder
breaks even.
Answer: FALSE
2) American style options can only be exercised on their expiration
dates.
Answer: FALSE
3) The party that accepts the legal obligation to stand behind the option
is the buyer of the contract.
Answer: FALSE

4) Listed options trade over-the-counter.


Answer: FALSE
5) Over-the-counter options are less structured than listed options and
are primarily purchased by individual investors.
Answer: FALSE
6) The majority of today's options are stock options traded primarily on
the CBOE and on AMEX.
Answer: TRUE
7) Technically, listed options expire on the Saturday following the third
Friday of the expiration month.
Answer: TRUE
8) European options can only be exercised on the expiration date but
can be sold to another investor on any trading day.
Answer: TRUE
9) Standardized options expire on the last business day of the
expiration month.
Answer: FALSE
10) The buyer of a listed American option has which of the following
rights?
I.
the right to change the expiration date
II. the right to change the strike price
III. the right to resell the option
IV. the right to let the option expire unexercised
A) I and III only
B) III and IV only
C) I, III and IV only
D) II, III and IV only
Answer: B
11) The writer of a put
A) accepts the obligation to sell a predetermined number of shares at a
predetermined price.
B) is betting the price of the underlying security will increase in value.
C) is hoping that the put will be in-the-money prior to expiration.
D) will pay the premium whether or not the option is exercised.
Answer: B
12) Which of the following is a possible official expiration date for a
standardized option contract?
A) Saturday, October 17
B) Monday, March 1
C) Friday, April 30
D) Wednesday, May 19
Answer: A
13) Which one of the following was the first listed exchange for stock
options in the United States?
A) Stock Index Board
B) Philadelphia Board of Trade
C) New York Stock Exchange
D) Chicago Board Options Exchange
Answer: D
14) The option premium is
A) the market price of the option.
B) the amount by which the stock price is expected to move before the
option expires.
C) the fee charged by the options exchanges for executing
transactions.
D) the difference between the strike price and the underlying price of
the security.
Answer: A
15) The strike price of a put option is the price
A) an investor must pay for the options contract.
B) of the underlying stock at the time that the options contract is
purchased.
C) the price at which the underlying stock can be sold.
D) the price at which the underlying stock can be bought.
Answer: C

16) Listed options


A) are traded directly between the buyer and the seller.
B) are rarely traded in the secondary markets.
C) have readily available price information.
D) are sold over the counter.
Answer: C
17) Stocks options that trade in the January cycle will have contracts
available that expire in
A) January, February, April, and July.
B) March, June, September, December.
C) January, February, March, and April.
D) each of the next 12 months.
Answer: A
18) The two provisions which investors should carefully consider when
evaluating stock options are the
A) strike price and the exchange ratio.
B) time until expiration and the strike price.
C) leverage ratio and the time to maturity.
D) premium and the discount.
Answer: B
19) For a call purchased on an organized security exchange, the strike
price specifies the
A) contractual price at which each of the shares of the underlying stock
can be bought.
B) prevailing market price of one share of the underlying stock.
C) cost of buying one option contact based on the value of the
underlying stock.
D) intrinsic value of the offsetting put.
Answer: A
20) For all practical purposes, listed stock options always expire
A) on the last business day of the expiration month.
B) on the first Monday of every calendar quarter.
C) on the third Friday of the expiration month.
D) three months from the date of the option purchase.
Answer: C
14.3 Learning Goal 3
1) The buyer of a put expects the price of the underlying stock to rise.
Answer: FALSE
2) The value of a call increases as the price of the underlying security
rises.
Answer: TRUE
3) The value of a put increases as the price of the underlying security
rises.
Answer: FALSE
4) The buyer of a put and the writer of the a both profit if the price of
the stock falls.
Answer: FALSE
5) The option premium is the price of the option.
Answer: TRUE
6) Investors buy options at the bid price and sell at the ask price.
Answer: FALSE
7) The price behavior of the underlying security is the primary
determinant of the price of an option.
Answer: TRUE
8) A put option has a strike price of $32. The current price of the stock
is $34. The put option is said to be "in-the-money."
Answer: FALSE
9) Grant purchased one call on XYZ stock at an exercise price of $25.
The market price of XYZ stock when Grant purchased the call was $24
a share. XYZ is currently priced at $30 a share. Grant paid $120 to buy
the call. How much profit will Grant make if he exercises the option
today and then sells the shares? Ignore all transaction-related costs.
A) $380

B) $480
C) $500
D) $600
Answer: A
10) Rex bought a put on Alpha stock with a strike price of $35 when
the market price of Alpha stock was $33 a share. Alpha is currently
selling at $34 a share. Which of the following statements are true given
this information?
I.
Rex's option is worth at least $100 today.
II. Rex's option is worthless today.
III. Rex's option has more value today than when he bought it.
IV. Rex's option has less value today than when he bought it.
A) I and III only
B) I and IV only
C) II and III only
D) II and IV only
Answer: B
11) One of the major disadvantages of options is
A) their lifespan.
B) their cost.
C) their lack of liquidity.
D) the risk to option buyers.
Answer: A
12) The most important factor affecting the market price of a put or call
is the
A) market interest rate.
B) expiration date.
C) price behavior of the underlying common stock.
D) price behavior of the corresponding warrant.
Answer: C
13) NZMA stock is currently selling for $128. Which of the following
options is "in-the-money"?
A) March 130 call
B) February 125 call
C) March 125 put
D) February 100 put
Answer: B
14) Which of the following affect the value of puts and calls written on
shares of common stock?
I.
price volatility of the underlying stock
II. current market price of the underlying stock
III. length of time until the option expiration date
IV. current market interest rate
A) I and II only
B) I, II and III only
C) II, III and IV only
D) I, II, III and IV
Answer: D
15) Lew paid $300 to purchase a call on Delta stock with a strike price
of $25. What does the market price of Delta have to be for Lew to
break-even on his option investment? Ignore transaction costs and
taxes.
A) $22
B) $25
C) $28
D) cannot be determined from the information provided
Answer: C
16) Andrea wrote a three-month call on Echo stock. The option cost
$200 and the strike price was $10. What does the market price of Echo
have to be for Andrea to break-even on this investment if the option is
exercised? Ignore transaction construed taxes.
A) $10
B) $12
C) $8
D) cannot be determined from the information provided
Answer: B
17) Jason purchased a six-month put on ABC stock at a cost of $100.
The strike price was $15. At what market price does Jason just breakeven on this investment? Ignore transaction costs and taxes.

A) $15
B) $16
C) $14
D) cannot be determined from the information provided
Answer: C
18) Jamie wrote a nine-month put on Beta stock. The strike price was
$25 and the market price at the time the option was written was $24.
The total price of the option contract was $150. At what market price
will Jamie just break-even on this investment? Ignore transaction costs
and taxes.
A) $23.50
B) $24.00
C) $25.00
D) $26.50
Answer: A
19) A put has fundamental value as long as
A) the market price of the underlying financial asset has a positive
value.
B) the market price of the underlying financial asset is less than the
strike price.
C) the strike price of the put is greater than the time premium of the
put.
D) the strike price of the put is less than the market value of the
underlying asset.
Answer: B
20) What is the fundamental value of a call with a strike price of $30
and a market price of $33?
A) -$300
B) -$3
C) $3
D) $300
Answer: D
21) Which of the following represent in-the-money options?
I.
a call when the market price exceeds the strike price
II. a call when the strike price exceeds the market price
III. a put when the market price exceeds the strike price
IV. a put when the strike price exceeds the market price
A) I and III only
B) I and IV only
C) II and III only
D) II and IV only
Answer: B
22) What is the time premium of a put with a strike price of $25 when
the option price is $2 and the underlying common stock sells for $24?
A) $100
B) $200
C) $300
D) $400
Answer: A
23) What is the fundamental value of a put contract with a strike price
of $25 when the option price is $1.50 and the underlying common
stock sells for $26?
A) $150
B) $100
C) $0.00
D) -$100
Answer: C
24) Nowel Inc. stock is currently priced at $42. The present value of
the strike price of a call option on this stock is $44. Probability one, as
calculated by the Black Scholes option pricing model is .6541;
probability 2 is .3722. The value of this option as calculated by BlackScholes is
A) $(2.00).
B) $11.10.
C) $2.000.
D) $10.71.
Answer: B
25) Which of the following increase(s) the time premium of a call
option?
I.
a market price that exceeds the strike price

II. increasing volatility in the market price of the underlying security


III. decreasing market interest rates
IV. decreasing the time to option expiration
A) II only
B) I and II only
C) III and IV only
D) II and III only
Answer: A
26) Which of the following variables are part of the Black-Scholes
option pricing model?
I.
the market price of the underlying stock
II.
the volatility of the underlying security
III.
the strike price of the option
IV.
the risk-free rate of interest
V.
the beta of the underlying security
VI. the time remaining before the option expires
A) I, II, IV and VI only
B) I, II and III only
C) I, II, III, IV and VI only
D) I, II, III, IV, V and VI
Answer: C

B) write a put
C) buy a put
D) sell a put
Answer: C
14) Roselle paid $250 to buy one put option with a strike price of $35.
What is the maximum profit Roselle can earn on her option contract?
A) $100
B) $350
C) $3,250
D) Her profit potential is unlimited.
Answer: C
15) The price of ABC stock is currently $42 per share, but in six
months you expect it to rise to $50. ABC does not pay a dividend. You
buy a six-month call on ABC, with a strike price of $45. The option cost
$200. What holding period return do you expect on this call? Ignore
transaction costs and taxes.
A) 150%
B) 200%
C) 250%
D) 300%
Answer: A

14.4 Learning Goal 4


1) Options can provide a lot of price action for a limited dollar
investment.
Answer: TRUE
2) One of the primary advantages of options is the leverage they
provide.
Answer: TRUE
3) The maximum amount the buyer of a put can lose is the cost of the
option.
Answer: TRUE
4) The writer of a call option is theoretically exposed to an unlimited
loss.
Answer: TRUE
5) If you expect the price of a security to decline, you could buy a call
to protect your financial position.
Answer: FALSE
6) Once the call premium is recouped, the profit from a call is only
limited by the price increases of the underlying stock prior to the
contract expiration.
Answer: TRUE
7) If a stock price does not rise or fall by the amount of the option
premium, the option will not be exercised.
Answer: FALSE
8) The longer the time to expiration, the lower the option time premium
tends to be.
Answer: FALSE
9) The maximum loss that can be incurred as the buyer of an option is
the amount of the option premium.
Answer: TRUE
10) Option writing can be very profitable because the majority of
options are never exercised.
Answer: TRUE
11) A naked option is a conservative investment with limited risk.
Answer: FALSE
12) An option straddle is the simultaneous purchase (or sale) of both a
put and a call option on the same underlying security.
Answer: TRUE
13) Kyle believes the price of Ajax stock is about to decrease. If he
wants to profit from the decline in price, he should ________ on Ajax
stock.
A) buy a call

16) Tiffany would like to own shares of Blackwood, Inc. but only if she
can acquire them at a total cost of $30 a share or less. Blackwood is
currently trading at $31.76. Cynthia should ________ with a strike
price of $30. Ignore transaction costs.
A) buy a call
B) buy a put
C) write a call
D) write a put
Answer: A
17) Fred bought 600 shares of Edgewood stock at a price of $19. The
stock is currently selling for $53 a share. To protect his profits, Fred
should buy
A) 600 call options with a strike price of $55.
B) 600 put options with a strike price of $50.
C) 6 call options with a strike price of $55.
D) 6 put options with a strike price of $50.
Answer: D
18) Shares of Lakewood, Inc. are currently selling for $52.63. You
believe the stock will decline in price ranging from $30 to $32 in the
next few months. Which of the following strategies will allow you to
profit if your prediction is correct?
I.
short the stock
II. buy a call at 50
III. write a call at 55
IV. buy a put at 45
A) II and IV only
B) I and III only
C) III and IV only
D) I, III and IV only
Answer: D
19) In January, JB stock was selling for $50 per share. When the calls
and the puts with a strike price of $45 expired on March 20, JB was
selling at $46. Which investors made a profit?
I.
the writer of the call
II. the buyer of the call
III. the writer of the put
IV. the buyer of the put
A) II and III
B) I and III
C) only III
D) II and IV
Answer: B
20) In nearly all cases, the purpose of a hedge is to
A) reduce or eliminate risk.
B) make a very high profit in an extremely short time frame.
C) speculate on a downward drop in a general market index.
D) speculate on an upward movement in a given currency.
Answer: A
21) Which one of the following actions would be the most appropriate

hedge to a short sale of common stock?


A) sale of a call
B) purchase of a call
C) sale of a put
D) purchase of a put
Answer: B
22) Steve bought 300 shares of stock at a price of $20 per share. The
price of the stock then went up to $33 per share so Steve decided to
hedge his position by purchasing 3 puts at a cost of $120 each. The
puts have an exercise price of 30. One week prior to the expiration of
the puts, the price of the stock was at $22 per share. If Steve closed
out all of his positions at that time, he would have earned a net profit of
A) $200.
B) $240.
C) $2,640.
D) $3,000.
Answer: C
23) Allison bought 100 shares of MIKO, Inc. stock at a price of $35 a
share. In addition, she bought a 35 put on MIKO at a cost of $125.
Which of the following are true about Allison's position from now until
the option expiration date?
I.
Her maximum loss is $3,625.
II. Her maximum loss is $125.
III. Her minimum gain is $125.
IV. Her maximum profit is unlimited.
A) I and IV only
B) II and III only
C) II and IV only
D) II, III and IV only
Answer: C
24) The purchase of a June 25 call on XXO stock and the sale of a
June 30 call on XXO stock is known as a
A) long straddle.
B) short straddle.
C) vertical spread.
D) horizontal spread.
Answer: C
25) Mathew simultaneously sold a July 40 put on ZXY stock for $200
and bought a July 35 put for $75. His maximum loss is ________ and
his maximum gain is ________.
A) $375, $125
B) $375, unlimited
C) $500, $125
D) $275, $125
Answer: A
26) The purchase of a June 25 call on XXO stock and the sale of a
June 30 call on XXO stock is known as a
A) long straddle.
B) short straddle.
C) vertical spread.
D) horizontal spread.
Answer: C
27) A long straddle
A) consists of selling and writing an equal number of puts and calls
with different strike prices but the same expiration date and the same
underlying security.
B) is a strategy based on the expectation that the price of the
underlying security will be relatively constant.
C) consists of buying a call at one strike price and then writing a call at
a higher strike price.
D) is a strategy that produces profits when the price of the underlying
security moves significantly in either direction.
Answer: D
14.5 Learning Goal 5
1) For the writer of in-the-money covered calls , losses on the options
contract will be nullified by gains on the stock.
Answer: TRUE
2) Writing covered calls may result in a profit to the writer even if the
stock price does not change.

Answer: TRUE
3) Writing covered calls protects the writer from losses if the price of
the underlying stock declines.
Answer: FALSE
4) Covered call writers have unlimited loss exposure as well as
unlimited profit potential.
Answer: FALSE
5) Matt owns 500 shares of IKM stock. The market price of IKM is
$51.74. Matt just sold five calls on IKM with a strike price of $50. This
is known as
A) writing a naked call.
B) writing a covered call.
C) creating a naked cover.
D) covering a short position.
Answer: B
6) Justin owns 400 shares of ORNG stock which he bought 10 months
ago at $20 per share and has now risen to $35 per share. He is afraid
the stock price will fall before he has owned it for a full year, but wants
to postpone realizing profits on the stock for several months, when it
will become a long-term rather than short-term gain. He can protect
his profit and avoid the short-term capital gains rate by
A) writing covered calls.
B) writing puts.
C) buying puts.
D) buying calls.
Answer: C
7) Bill owns 200 shares of EG stock. In November, the market price of
EG was $15.45. Bill sold two March 16 calls on EG for $246. Between
November and March, EG stock fluctuated between $14.75 and
$15.85. EG paid a quarterly dividend of $0.40 per share on January
31. Over the November-March period, Bill earned
A) $80.
B) $(176).
C) $336.
D) $256.
Answer: C
8) Mary wrote a 40 call on ABC stock at a price of $275. She does not
own any shares of ABC. Mary has
I.
limited her losses to $275.
II. unlimited loss potential.
III. limited her gains to $275.
IV. unlimited profit potential.
A) I and IV only
B) II and III only
C) I and III only
D) II and IV only
Answer: B
9) The writer of a covered call has taken a(n)
A) conservative investment position with unlimited potential profits.
B) conservative investment position with limited profits.
C) aggressive position with limited losses and unlimited potential
profits.
D) aggressive position with potentially unlimited profits or losses.
Answer: B
14.6 Learning Goal 6
1) While stock index options can be used to play the market as a
whole, they are also effective in protecting equity portfolios against
falling markets.
Answer: TRUE
2) To exercise a call option on the Dow Jones Industrial Average, an
investor would need to actually buy all 30 stocks at the strike price.
Answer: FALSE
3) The value of an interest rate call option increases when interest
rates fall.
Answer: TRUE

4) Put and call index options are available only the S&P 500 Index, the
NASDAQ 100 and the DJIA.
Answer: FALSE
5) Long-term Equity AnticiPation Securities (LEAPS) are a form of
option that gives the holder the right to buy newly issued shares of
stock directly from the issuing corporation.
Answer: FALSE
6) If the S&P 500 index is at 1,461, then the cash value of an S&P 500
index option is
A) $14.61.
B) $1,461.
C) $14,610.
D) $146,100.
Answer: D
7) One could temporarily protect profits on a highly diversified portfolio
of large company stocks by
A) selling S&P 500 Index put options.
B) buying S&P 500 Index put options.
C) buying S&P 500 Index call options.
D) selling S&P 500 Index call options.
Answer: B
8) Bob's DJIA Index option had a strike price of 125. When he
exercised the option, the Dow was at 13,050.
A) Bob received $5,500 from the writer of the contract.
B) Bob paid $550 to the writer of the contract.
C) Bob received $550 from the writer of the contract.
D) Bob received $55,000 from the writer of the contract.
Answer: C
9) The premium on a stock index call would be expected to increase as
the
A) market becomes more volatile.
B) option life nears expiration.
C) index price falls further below the strike price.
D) underlying securities stabilize in value.
Answer: A
10) ETF options are settled in
A) cash.
B) ETF shares.
C) share of the companies in the index.
D) The writer has the choice of settling in either cash or ETF shares.
Answer: B
11) Anthony is confident that shares of SolarTech will greatly increase
in value, but thinks that it may be a year or more before that happens.
He should buy
A) ETF calls.
B) LEAP puts.
C) LEAP calls.
D) Index calls.
Answer: C
12) Stock index options can be used for which of the following
investment purposes?
I.
protect a portfolio from market declines

II. speculate on the price appreciation of a particular common stock


III. take advantage of a leverage opportunity
IV. create a portfolio hedge
A) I and IV only
B) II and IV only
C) I and III only
D) I, III and IV only
Answer: D
13) The value of an interest rate call option
A) varies directly with the price of the underlying corporate bond.
B) increases when the yield on the underlying Treasury security rises.
C) is based on the market price of U. S. Treasury securities.
D) decreases when the price of U.S. Treasuries decreases.
Answer: B
14) If the Canadian dollar became stronger relative to the U.S. dollar,
the price of
A) a call option on the Canadian dollar will increase.
B) a put option on the Canadian dollar will increase.
C) a call option on the Canadian dollar will decrease.
D) both the call and the put options on the Canadian dollar will
decrease.
Answer: A
15) The currency option strike price of 163 means that
A) $1 is worth 1.63 units of the foreign currency.
B) $1 is worth 163 units of the foreign currency.
C) one unit of the foreign currency is worth $1.63.
D) one unit of the foreign currency is worth $163.
Answer: C
16) Which of the following statements concerning Long-term Equity
AnticiPation Securities (LEAPS) is correct?
A) LEAPS are traded solely in the over-the-counter market.
B) LEAPS are options that are available only on individual common
stocks.
C) LEAPS typically have a higher quoted price than that of a regular
option.
D) LEAPS generally have a longer life than a warrant.
Answer: C
17) Which of the following methods might be used to protect a profit on
a diversified portfolio of stocks?
A) Buy S&P 500 Index put options.
B) Buy put options on a S & P 500 based ETF.
C) Write S&P 500 Index put options.
D) Either A or B, but not C.
Answer: D
18) An investor who exercises a call option on a S&P 500 ETF will
A) purchase ETF shares at the strike price.
B) receive a cash settlement equivalent to the difference between the
strike price and the current level of the index.
C) receive a cash settlement equivalent to the difference between the
strike price and 100 times the current level of the index.
D) receive a cash settlement equivalent to the difference between the
strike price and the current price of the ETF.
Answer: C

Вам также может понравиться